Is the Product of a Null Sequence and a Bounded Sequence Always Null?

Click For Summary
The discussion centers on proving that the product of a null sequence (a_n) and a bounded sequence (b_n) is also a null sequence. It is established that since b_n is bounded, there exists a real number H such that |b_n| ≤ H. Given that a_n converges to 0, for any ε > 0, there exists an N such that for n > N, |a_n| < ε/H. The proof concludes that for n > N, |a_n b_n| < ε, demonstrating that a_n b_n converges to 0. Concerns about the negativity of H and the implications for the proof are addressed, clarifying that |a_n| cannot be negative.
phospho
Messages
250
Reaction score
0
Prove that if (a_n) is a null sequence and (b_n) is a bounded sequence then the sequence (a_nb_n) is null:

from definitions if b_n is bounded then ## \exists H \in \mathbb{R} ## s.t. ## |b_n| \leq H ## if a_n is a null sequence it converges to 0 (from my book), i.e. given ## \epsilon ' > 0 ## ## \exists N \in \mathbb{R} ## s.t. n>N ## \Rightarrow |a_n| < \epsilon ' ## set ## e' = \dfrac{\epsilon}{|H|} ## then for ## n > N \Rightarrow |a_nb_n | < \epsilon ' |H| = \epsilon ## i.e. a_nb_n also converges to 0.

I'm slightly worried about this, if H is negative, then can we just multiply ## |a_n| ## and ## |b_n| ## as afaik from the axioms if ## a>0 ## ## x>y## then ## xa>ya ## and I'm not sure if H is negative or if |a_n| is negative if we can simply just multiply the two inequalities. Any help, explaining this, thanks.
 
Physics news on Phys.org
phospho said:
Prove that if (a_n) is a null sequence and (b_n) is a bounded sequence then the sequence (a_nb_n) is null:

from definitions if b_n is bounded then ## \exists H \in \mathbb{R} ## s.t. ## |b_n| \leq H ## if a_n is a null sequence it converges to 0 (from my book), i.e. given ## \epsilon ' > 0 ## ## \exists N \in \mathbb{R} ## s.t. n>N ## \Rightarrow |a_n| < \epsilon ' ## set ## e' = \dfrac{\epsilon}{|H|} ## then for ## n > N \Rightarrow |a_nb_n | < \epsilon ' |H| = \epsilon ## i.e. a_nb_n also converges to 0.

I'm slightly worried about this, if H is negative, then can we just multiply ## |a_n| ## and ## |b_n| ## as afaik from the axioms if ## a>0 ## ## x>y## then ## xa>ya ## and I'm not sure if H is negative or if |a_n| is negative if we can simply just multiply the two inequalities. Any help, explaining this, thanks.

How can ##|a_n|## be negative?
 
phospho said:
Prove that if (a_n) is a null sequence and (b_n) is a bounded sequence then the sequence (a_nb_n) is null:

from definitions if b_n is bounded then ## \exists H \in \mathbb{R} ## s.t. ## |b_n| \leq H ## if a_n is a null sequence it converges to 0 (from my book), i.e. given ## \epsilon ' > 0 ## ## \exists N \in \mathbb{R} ## s.t. n>N ## \Rightarrow |a_n| < \epsilon ' ## set ## e' = \dfrac{\epsilon}{|H|} ## then for ## n > N \Rightarrow |a_nb_n | < \epsilon ' |H| = \epsilon ## i.e. a_nb_n also converges to 0.

Since your proof is correct, I think I can get away with showing you how to write it up a bit nicer:

Since ##\{b_n\}## is bounded there is ##H \in \mathbb{R} ## such that ##|b_n|\le H## for all natural numbers ##n##. Suppose ##\epsilon > 0##. Since ##a_n\to 0## there exists ##N\in \mathbb R## such that if ##n>N## we have ##|a_n| < \frac \epsilon H##. Now if ##n>N## then we have ##|a_nb_n-0| = |a_n||b_n| < \frac \epsilon H\cdot H =\epsilon##. Hence ##a_nb_n\to 0##.
 
LCKurtz said:
How can ##|a_n|## be negative?

ok ##|a_n| < \epsilon ## this is the distance from a_n to 0 which is less than epsilon so I guess it can't be negative. But what if ## |b_n| \leq H ## and H is negative? take e.g. ## |b_n| \leq -1 ## then ## 1 \leq b_n \leq -1 ##
 
phospho said:
take e.g. ## |b_n| \leq -1 ## then ## 1 \leq b_n \leq -1 ##

Doesn't ##1\le -1## bother you? Or ##|b_n|\le -1##?
 
LCKurtz said:
Doesn't ##1\le -1## bother you? Or ##|b_n|\le -1##?

ha - yeah, thanks.

Also thanks for writing up a more concise proof!
 
Question: A clock's minute hand has length 4 and its hour hand has length 3. What is the distance between the tips at the moment when it is increasing most rapidly?(Putnam Exam Question) Answer: Making assumption that both the hands moves at constant angular velocities, the answer is ## \sqrt{7} .## But don't you think this assumption is somewhat doubtful and wrong?

Similar threads

  • · Replies 8 ·
Replies
8
Views
3K
Replies
5
Views
2K
  • · Replies 1 ·
Replies
1
Views
2K
Replies
4
Views
2K
  • · Replies 1 ·
Replies
1
Views
1K
  • · Replies 2 ·
Replies
2
Views
2K
  • · Replies 7 ·
Replies
7
Views
2K
  • · Replies 28 ·
Replies
28
Views
3K
  • · Replies 13 ·
Replies
13
Views
2K
  • · Replies 14 ·
Replies
14
Views
3K